Abstract Algebra: List elements of Subgroup

Click For Summary
The discussion revolves around identifying the elements of the subgroups <3> and <7> in U(20), which consists of the units {1, 3, 7, 9, 11, 13, 17, 19}. The original poster mistakenly considered addition as the operation instead of multiplication modulo 20. It is clarified that the cyclic subgroup generated by 3 is not equal to U(20) and should include elements derived from multiplying 3 and 7. The correct elements of the subgroups are identified as <3> = <7> = {1, 3, 7, 9}. The conversation concludes with the poster acknowledging the error in their approach.
srfriggen
Messages
304
Reaction score
7

Homework Statement



List the elements of the subgroups <3> and <7> in U(20).



Homework Equations





The Attempt at a Solution



U(20)= {1, 3, 7, 9, 11, 13, 17, 19} = <3> = <7>.



So basically I have that the common elements of, <3> and <7> and U(20), under + modulo 20, are all the elements in U(20). Other than calculating this all out, I reasoned it to be true since 3 and 7 are both coprime to 20.

The answer key in the book has <3>=<7>= {3,9,7,1}.

I believe this is a typo, and that either the answer key is incorrect, or U(20) should be U(10).

Can someone please confirm my suspicion or let me know if I am missing something?
 
Physics news on Phys.org
Remember, U(n) is the group of units mod n and the operation is multiplication mod n. You are right that U(20)={1,3,7,9,11,13,17,19}. But, the cyclic subgroup generated by 3 is certainly not U(20). We note that 3(3) mod 20 = 9. Then, 3(9) mod 20 = 7 mod 20. [Remember, 3(9) = 27 mod 20 = 7]. Now, so far we see that <3>={3,9,7,...}. Next, multiply 7 by 3 and mod out by 20. What do we get? =)
 
jmjlt88 said:
Remember, U(n) is the group of units mod n and the operation is multiplication mod n. You are right that U(20)={1,3,7,9,11,13,17,19}. But, the cyclic subgroup generated by 3 is certainly not U(20). We note that 3(3) mod 20 = 9. Then, 3(9) mod 20 = 7 mod 20. [Remember, 3(9) = 27 mod 20 = 7]. Now, so far we see that <3>={3,9,7,...}. Next, multiply 7 by 3 and mod out by 20. What do we get? =)

I see. I was using addition as the operation. Thank you!
 
No problem. =)
 
Question: A clock's minute hand has length 4 and its hour hand has length 3. What is the distance between the tips at the moment when it is increasing most rapidly?(Putnam Exam Question) Answer: Making assumption that both the hands moves at constant angular velocities, the answer is ## \sqrt{7} .## But don't you think this assumption is somewhat doubtful and wrong?

Similar threads

  • · Replies 1 ·
Replies
1
Views
2K
Replies
14
Views
2K
  • · Replies 4 ·
Replies
4
Views
3K
  • · Replies 7 ·
Replies
7
Views
1K
  • · Replies 1 ·
Replies
1
Views
1K
  • · Replies 22 ·
Replies
22
Views
3K
Replies
5
Views
2K
  • · Replies 1 ·
Replies
1
Views
2K
  • · Replies 7 ·
Replies
7
Views
3K
  • · Replies 28 ·
Replies
28
Views
5K